Soluções problemas da semana 8

Iniciante: Seja S o menor conjunto de inteiros positivos tal que:

a) 2 está em S

b) Se n^2 está em S, n está em S

c) Se n está em S, (n + 5)^2 está em S

Quais inteiros positivos não estão em S?

(Escrito por Luca Zanardi)

Conhecimentos Prévios Necessários:

Primeiramente, note que, se n está em S, (n+5)^2 está em S, e assim, $n+5$ também estará obrigatoriamente. Dessa forma, é fácil ver que, se n está em S, 5k+n também estará em S, para todo k inteiro não negativo. Como 2 está em S, todo número inteiro positivo congruente a 2 \pmod{5} também está.

Agora, perceba que (2+5)^2 = 49 está em S, e X = (49+5)^2 \equiv (-1)^2 \equiv 1 \pmod{5} está em S, portanto, todo número maior que X congruente a 1 está em S. Em particular, os números 3^{(2^n)}, 4^{(2^n)}, 6^{(2^n)} também estão, para algum n suficientemente grande, logo, 3, 4, 6 também estão. Assim sabemos que todo número congruente a 1, 2, 3, 4 \pmod{5} maior que 1 está em S. Com isso, veja que S = \{ x | x  data-recalc-dims= 1, 5 \nmid x \}" /> é um conjunto válido. Portanto, nossa resposta é 1 e todos os números divisíveis por 5.

INTERMEDIÁRIO: Determine todos os pares de inteiros positivos (a,b) tais que  a \mid b \mid a^2 \mid b^3 \mid a^4 \mid b^5 \mid \cdots

(Escrito por João Gilberti)

Conhecimentos Prévios Necessários:

  • Expoentes em Primos
  • Teorema do Sanduíche

Temos que, \forall k \in \mathbb{N}, temos que a^{2k} \mid b^{2k+1} \mid a^{2k+2}, portanto, tome p um primo qualquer,

 \nu_p(a^{2k}) \le \nu_p (b^{2k+1}) \le \nu_p (a^{2k+2}) \Longrightarrow 2k \nu_p(a) \le (2k+1) \nu_p(b) \le (2k+2) \nu_p(a)  \Longrightarrow \frac{2k}{2k+1} \le \frac{\nu_p(b)}{\nu_p(a)} \le \frac{2k+2}{2k+1}

Para todo k \in \mathbb{N}, então, veja que \lim_{k \rightarrow \infty} \frac{2k}{2k+1} = \lim_{k \rightarrow \infty} 1 - \frac{1}{2k+1} = 1, e que \lim_{k \rightarrow \infty} \frac{2k+2}{2k+1} = \lim_{k \rightarrow \infty} 1 + \frac{1}{2k+1} = 1, logo, pelo teorema do sanduíche, temos que

\lim_{k \rightarrow \infty} \frac{2k}{2k+1} \le \lim_{k \rightarrow \infty} \frac{\nu_p(b)}{\nu_p(a)} \le \lim_{k \rightarrow \infty} \frac{2k+2}{2k+1} \Longrightarrow 1 \le \frac{\nu_p(b)}{\nu_p(a)}\le 1 \Longrightarrow \nu_p(b) = \nu_p(a)

Para todo primo p, ou seja, a fatoração em primos de a, b são as mesmas, então, a = b.

AVANÇADO: Seja p_i com i = 1, 2, \cdots , k a sequência dos números primos (Por exemplo, p_1 = 2, p_2 = 3, p_3 = 5 \cdots). Seja N = p_1 p_2 \cdots p_k. Prove que no conjunto  \{ 1, 2, \cdots , n \} existem exatamente \frac{N}{2} números que são divisíveis por uma quantidade impar de p_i.

(Escrita por Leonardo do Carmo)

Solução